I am studying the second chapter of Peskin and Schroeder's QFT text. In equation 2.27 and 2.28, the book defines the field operators:
ϕ(x)=∫d3p(2π)31√2wp(ap+a†−p)eipxπ(x)=∫d3p(2π)3(−i)√wp2(ap−a†−p)eipx
The momentum operator is then calculated in equation 2.33. However, my own derivation gives a different answer. I am reproducing my steps hoping that someone will be able to find where I went wrong. Starting with the definition of the momentum (conserved charge of spatial translations):
P=−∫d3xπ(x)∇ϕ(x)P=−∫d3x[∫d3p(2π)3(−i)√wp2(ap−a†−p)eipx][∫d3p′(2π)31√2wp′(ap′+a†−p′)∇eip′x]P=−∫∫d3p(2π)3d3p′(2π)3∫d3x(−i2)p′ei(p+p′)x[√wp2(ap−a†−p)][1√2wp′(ap′+a†−p′)]P=−∫∫d3p(2π)3d3p′(2π)3p′(2π)3δ(p+p′)√wp2(ap−a†−p)1√2wp′(ap′+a†−p′)P=−∫∫d3p(2π)3(−p)√wp2(ap−a†−p)1√2w−p(a−p+a†p) Since wp=w−p=|p|2+m2, we get P=−∫d3p2(2π)3(−p)√wpwp(ap−a†−p)(a−p+a†p)P=∫d3p(2π)3p2(ap−a†−p)(a−p+a†p)P=∫d3p(2π)3p2[apa−p+apa†p−a†−pa−p−a†−pa†p]P=∫d3p(2π)3p2[apa−p−a†−pa†p]+∫d3p(2π)3p2[apa†p−a†−pa−p]
The first integral is odd with respect to p, and vanishes. For the second term, we can formally prove that a†−pa−p=a†pap, but we can also argue that from noting that this operator pair creates a particle but then destroys it, with any possible constants only depending on the magnitude of \mathbf{p}. This line of reasoning gives us:
P=∫d3p(2π)3p2(apa†p−a†pap)=∫d3p(2π)3p2[ap,a†p]
The commutator here is proportional to the delta function, and hence this expression doesn't match what Peskin & Schroeder, and other QFT books have, i.e., P=∫d3p(2π)3pa†pap
UPDATE: I realized later that my assumption that a†−pa−p=a†pap was wrong. When I was trying to prove this using the expansion of the ladder operators in terms of ϕ(x) and π(x) I was making an algebra error.
Answer
From P=∫d3p(2π)3p2(apa†p−a†−pa−p) you actually have P=∫d3p(2π)3(p2apa†p−p2a†−pa−p)=∫d3p(2π)3(p2apa†p−−p2a†pap)=∫d3p(2π)3(p2a†pap+p2a†pap)+renormalization term. Dropping the infinite renormalization term due to δ(0) P=∫d3p(2π)3pa†pap.
No comments:
Post a Comment